LSAT and Law School Admissions Forum

Get expert LSAT preparation and law school admissions advice from PowerScore Test Preparation.

 GLMDYP
  • Posts: 100
  • Joined: Aug 19, 2013
|
#10418
Hi Powerscore!
The right answer for this question is (A), but I cannot find (A) have demonstrated that the doctor's second reason does not cancel out the first. Can you show me how and why?
Thanks!
 David Boyle
PowerScore Staff
  • PowerScore Staff
  • Posts: 836
  • Joined: Jun 07, 2013
|
#10474
GLMDYP wrote:Hi Powerscore!
The right answer for this question is (A), but I cannot find (A) have demonstrated that the doctor's second reason does not cancel out the first. Can you show me how and why?
Thanks!
Hello,

It does not cancel out the first reason because if the records have to be ready, then they'll, e.g., be clogging up the space just behind the counter rather than being able to be retrieved (or not retrieved) in some dusty warehouse 25 miles away.

David
 LustingFor!L
  • Posts: 80
  • Joined: Aug 27, 2016
|
#34619
Is this a common question stem for justify the conclusion? If it was not in chapter six of my question type training book, justify the conclusion, I do not think I would have recognized it as this question type. I know "if" is a sufficient question stem indicator, which is common in justify the conclusion questions stems. Also, I realize this is from a 1991 test, but I know that lsat test makers like to revert to old patterns to throw people off.

Just kinda thrown off by this...
User avatar
 Dave Killoran
PowerScore Staff
  • PowerScore Staff
  • Posts: 5853
  • Joined: Mar 25, 2011
|
#34641
Hi Lusting,

Good question! This is one of the most unusual question stems on record, and in the years since it was first presented, there has not been another question like it. So, you've found one that is truly unique!

If it helps, you were not alone in being thrown off by this stem—most students who saw this question were thrown for a loop by it and had a hard time understanding what was being said. In those instances where the question stem is unusual or doesn't make sense, the advice I always give is to focus less on classifying the question, and to instead focus on identifying exactly what it is that the question stem wants you to do. If you are able to do that, you will at least be heading in the right direction for finding the correct answer. In this case, you are looking to help out the first reason, and to show that the first reason would occur regardless of what happens with the second reason. That task is the same as justifying the first reason.

One question that might come up is, what's the value of classifying question stems if when you encounter a strange one I tell you to focus on meaning and less on classification? The answer is speed. The majority of question stems you see will be standard. If you can classify those instantly, you save time and energy over the long haul. Of course, once in a while you'll see a strange question, so in those cases just make sure you get the basic idea as described above. That allows you to move quickly in most cases but then be covered in those rare instances where you get a strange stem like the one in this question.

Please let me know if that helps. Thanks!
 LustingFor!L
  • Posts: 80
  • Joined: Aug 27, 2016
|
#34660
Thank you, Dave! That helps a lot :-D
 biskam
  • Posts: 124
  • Joined: Aug 18, 2017
|
#40151
David Boyle wrote:
GLMDYP wrote:Hi Powerscore!
The right answer for this question is (A), but I cannot find (A) have demonstrated that the doctor's second reason does not cancel out the first. Can you show me how and why?
Thanks!
Hello,

It does not cancel out the first reason because if the records have to be ready, then they'll, e.g., be clogging up the space just behind the counter rather than being able to be retrieved (or not retrieved) in some dusty warehouse 25 miles away.

David
Still struggling with how A is right. A strengthens the fact that giving patients access will be time wasting how? If the records are ready immediately, wouldn't that be not time wasting bc staff doesn't have to waste time to retrieve the records?

I'm struggling with what the right answer is supposed to do relative to the first statement and how A succeeds.

Thank you!
 biskam
  • Posts: 124
  • Joined: Aug 18, 2017
|
#40152
following up to show the status of my thought process:

Going into the question, it says find an answer that establishes that the first reason will occur regardless, that giving patients access will be time wasting... but it seems the new law in A will save time, not waste time.
 Eric Ockert
PowerScore Staff
  • PowerScore Staff
  • Posts: 164
  • Joined: Sep 28, 2011
|
#40170
Hi biskam!

Theoretically, the second reason seems like it does cancel out the first reason. If no patients ask for their records, how is this going to waste the staff's time? It would seem as though they aren't going to have to do anything.

The question stem is asking for a fact that, if true, would prove that even if patients don't ask for the files (the second reason is true), this legal right will cause a waste in time. Answer choice (A) establishes that medical staff has to retrieve the files whether or not a patient actually requests the file. So even if patients don't ask for them, the medical staff will have already had to spend the time fetching the files making them ready to be produced.
 akanshalsat
  • Posts: 104
  • Joined: Dec 20, 2017
|
#42551
Hello, I'm going through LRTT chapter 6 - justify the conclusion section, I've noticed that a lot of these questions aren't explained properly using the mechanistic method even though the companion Bible chapter focused heavily on conditioning us to use this mostly full-proof method. I do not understand how the mechanistic approach would be used for a question like this, for example because, the word "records" is written both in the conclusion and the premises, and information that is common to both conclusion and premises, according to the rule, is not supposed to be found in the right answer? I'm really confused and frustrated because I've gotten a majority of these questions incorrect because I could not figure out how to approach them with the Mechanistic rule.
User avatar
 Jonathan Evans
PowerScore Staff
  • PowerScore Staff
  • Posts: 726
  • Joined: Jun 09, 2016
|
#42615
Hi, Akansha,

Excellent questions. This is a highly unusual justify problem in that it asks you not to justify the main conclusion but to justify another statement identified within the question stem. To wit:
  • We must prove that the doctor's second statement DOES NOT contradict the first.
We must use the information in the stimulus to establish that the truth of the second statement does not imply the falsehood of the first statement.

Let's interpret this through a mechanistic approach. Per the mechanistic approach, remember that we are looking primarily for a "Rogue Element," something that appears in the conclusion that does not appear in the premises. Elements common to the conclusion and premises usually do not appear in the correct answer, but common elements certainly can appear in both, depending on how the question is structured.

Because of this question task, we need to treat the non-falsehood of the first condition as the necessary outcome or conclusion. That is:
  • IT COULD BE TRUE THAT 1) giving ppl access to medical records will be time-wasting by forcing staff to retrieve and return files and not do other stuff.
Thus, mechanistically/formally we could write,
  • Given the information in the 2nd statement and information in the correct response, IT COULD BE TRUE THAT 1) giving ppl access to medical records will be time-wasting by forcing staff to retrieve and return files and not do other stuff.
Let's make explicit the information in the 2nd statement:
  • 2) No patients will ask for access to their records.
Thus, all together:
  • If 2) No patients will ask for access to their records & [Information in answer choice] :arrow: IT COULD BE TRUE THAT 1) giving ppl access to medical records will be time-wasting by forcing staff to retrieve and return files and not do other stuff.
The Mechanistic Approach™ would imply finding a rogue element in this conclusion not present in the premise (statement 2).

What is discussed in the 1st reason that is not discussed in the 2nd reason?
  • The incongruity is the connection between "not asking for access [to medical records]" and "spending time retrieving [medical records]".
Thus, the credited response must bridge this gap. The correct answer must show that "not asking for access [to medical records]" DOES NOT imply staff will not "spend time retrieving [medical records]".

The information in answer choice (A) does address this gap. It shows that whether or not people ask for access [to medical records], the records must be ready immediately. Thus, irrespective of what patients do, the staff will always have to "spend time retrieving [medical records]".

Thus, we have a logically valid statement:
  • If 2) No patients will ask for access to their records & (A) Staff must retrieve records anyways :arrow: IT COULD BE TRUE THAT 1) giving ppl access to medical records will be time-wasting by forcing staff to retrieve and return files and not do other stuff.
Notice that this question is very similar to Resolve the Paradox tasks. We have to find a statement in the correct answer that would address both sides of this paradox. Explain how both could be possible:
  1. Giving ppl access to medical records will be time-wasting by forcing staff to retrieve and return files and not do other stuff.
  2. No patients will ask for access to their records.
Answer choice (A) addresses both sides of this paradox.

This is a very old and unusual question, but I hope the above explanation helps.

Please jump in on other Justify questions if you'd like a more conventional "Mechanistic Approach" explanation.

Get the most out of your LSAT Prep Plus subscription.

Analyze and track your performance with our Testing and Analytics Package.